Nur 207 - Respiratory - Saunders and Lewis

Ace your homework & exams now with Quizwiz!

Pneumothorax: *Diminished or absent breath sounds on the affected side.

Pneumothorax is characterized by restlessness, tachycardia, dyspnea, pain with respiration, asymmetrical chest expansion, and diminished or absent breath sounds on the affected side. Pneumothorax can cause increased airway pressure because of resistance to lung inflation.

Activity Intolerance

Pulse of 100 and deep respirations are expected adaptations to activity. Flushed skin is an expected response to activity; respirations will increase in depth rather than become slow. An increase in blood pressure is an expected response to activity; respirations probably will increase in depth and rate. ***Diaphoresis and tachypnea indicate that the client has exceeded tolerance for the activity.

Purpose of this type of breathing: *It promotes carbon dioxide elimination.

Pursed-lip breathing facilitates maximal expiration for clients with obstructive lung disease. This type of breathing allows better expiration by increasing airway pressure that keeps air passages open during exhalation.

A nurse is teaching a client with chronic airflow limitation (CAL) about positions that help breathing during dyspneic episodes. Which position, assumed by the client, would indicate that the client needs additional teaching regarding positioning? **Lying on his or her back in a low Fowler's position

Sitting is better than standing, whenever possible. If no chair is available, then leaning against a wall while standing allows accessory muscles to be used for breathing rather than posture control.

When planning appropriate nursing interventions for a patient with metastatic lung cancer and a 60-pack-per-year history of cigarette smoking, the nurse recognizes that the smoking has most likely decreased the patient's underlying respiratory defenses because of impairment of: **Mucociliary clearance

Smoking decreases the ciliary action in the tracheobronchial tree, resulting in impaired clearance of respiratory secretions and particles, chronic cough, and frequent respiratory infections.

A nurse administers oxygen at 2 L/min via nasal cannula to a client with emphysema. For which clinical indicators should the nurse closely observe the client? Select all that apply. *Drowsiness *Mental confusion

Some clients with chronic pulmonary obstructive disease (COPD) respond only to the chemical stimulus of low oxygen levels. Administration of high concentrations of oxygen to these individuals will eliminate the stimulus to breathe, leading to decreased respirations, lethargy, and drowsiness. Clients with COPD experience the Haldane effect; as hemoglobin molecules become more saturated with oxygen, they are unable to transport carbon dioxide out of the body, leading to hypercapnia. Increased levels of carbon dioxide depress the central nervous system, causing mental confusion and a lowered level of consciousness. Rising carbon dioxide levels cause lethargy rather than anxiety. Cyanosis is caused by excessive amounts of reduced oxyhemoglobin; because oxygen is being administered, cyanosis will not occur. ****High concentrations of oxygen may eliminate the stimulus to breathe, so the respiratory rate will decrease, not increase.

A client is diagnosed with a rib fracture and asks the nurse why strapping of the ribs is not being done. Which response by the nurse is most appropriate? **"That isn't done because people often would develop pneumonia from the constricting effect on the lungs."

Strapping of the ribs has a constricting effect on the ribs and on deep breathing and can actually increase the risk of atelectasis and pneumonia.

A nurse who is participating in a client care conference with other members of the health care team is discussing the condition of a client with acute respiratory distress syndrome (ARDS). The health care provider states that as a result of fluid in the alveoli, surfactant production is falling. The nurse understands that which is the natural consequence of insufficient surfactant?

**Collapse of alveoli and decreased compliance

A client who has been diagnosed with pleurisy tells the nurse that it is painful to inhale. The nurse responds that this is an expected finding owing to which physical response to this disorder?

**The inflamed pleurae cannot glide against each other as they normally do.

The nurse is changing the tracheostomy ties on a client with a tracheostomy and is assessing the security of the ties. Which method is used to ensure that the ties are not too tightly placed?

**The nurse places two fingers between the tie and the neck

A client who experiences frequent upper respiratory infections (URIs) asks the nurse why food does not seem to have any taste during illness. In formulating a response, the nurse understands that this effect is caused by which problem?

Blocked nasal passages impair the senses of smell and taste.

What should the nurse conclude is the most likely cause of the pleural effusion in a patient with lung cancer

Cancerous lesions in the pleural space increase the osmotic pressure, causing a shift of fluid to that space.

The nurse is caring for a group of clients on the clinical nursing unit. The nurse interprets that which of these clients is at most risk for the development of pulmonary embolism? **A 73-year-old woman who has just had pinning of a hip fracture.

Clients frequently at risk for pulmonary embolism include clients who are immobilized. This is especially true in the immobilized postoperative client.

A nurse is caring for a client with a chest tube drainage system and notes constant bubbling in the water seal chamber. Which nursing action is appropriate? **Notify the health care provider (HCP).

Constant bubbling occurring in the water seal chamber may indicate an air leak in the system. Among the options provided, the appropriate action is to notify the HCP.

A client with acquired immunodeficiency syndrome (AIDS) has histoplasmosis. The nurse should assess the client for which expected finding? **Dyspnea

Histoplasmosis is an opportunistic fungal infection that can occur in the client with acquired immunodeficiency syndrome (AIDS). The infection begins as a respiratory infection and can progress to disseminated infection. Typical signs and symptoms include fever, dyspnea, cough, and weight loss. Enlargement of the client's lymph nodes, liver, and spleen may occur as well.

The nurse evaluates that a patient is experiencing the expected beneficial effects of ipratropium (Atrovent) after noting which assessment finding? ** Increased peak flow readings

Ipratropium is a bronchodilator that should result in increased peak expiratory flow rates (PEFRs).

Huntington's disease

Neurodegenerative genetic disorder that affects muscle coordination and leads to mental decline and behavioral symptoms. The client with Huntington's disease experiences problems with motor skills such as swallowing and is at high risk for aspiration

The nurse is caring for a client with a respiratory disorder who is attempting to stop smoking. The health care provider has recommended nicotine (Nicorette) gum. When reviewing this treatment with the client, the nurse should provide which instruction to the client? **Hold the gum between the cheek and teeth periodically.

Nicotine gum should be chewed for 30-minute intervals with periods of holding the gum between the cheek and teeth; food and drink shoud be avoided 15 minutes before or during use.

Which is one of the first manifestations associated with tuberculosis? **Cough

One of the first pulmonary manifestations of tuberculosis is: a slight cough with the expectoration of mucoid sputum.

The primary reason safety precautions should be implemented with oxygen is because:

Oxygen is necessary for the production of fire. Oxygen does not burn; it supports combustion

The nurse is doing volunteer work in a homeless shelter. The nurse should monitor the individuals for which initial signs/symptoms of tuberculosis (TB)? Select all that apply.

The symptoms of TB include a slight *morning cough, *fatigue, *lethargy, and *low-grade fever.

Immediately after removal of the endotracheal tube: **Stridor

Recall that the primary concern after removal of an artificial airway is the client's inability to maintain a patent airway and breathe independently. Stridor indicates laryngeal edema and possible airway obstruction, it is the symptom that must be reported immediately.

Stridor

The sound is a stridor, which is a high pitched crowing sound that is often heard after traumatic intubations, and is usually heard at the larynx. Heard on inspiration, the sound often is audible without a stethoscope. A stridor is not heard at the nasal cavity, trachea, or pharynx.

A chest x-ray report for a client indicates the presence of a left apical pneumothorax. The nurse would assess the status of breath sounds in that area by placing the stethoscope in which location? **Just under the left clavicle

A chest x-ray report for a client indicates the presence of a left apical pneumothorax. The nurse would assess the status of breath sounds in that area by placing the stethoscope in which location?Rationale: The apex of the lung is the rounded, uppermost part of the lung. Therefore the nurse would place the stethoscope just under the left clavicle. All of the other options are incorrect locations for assessing the left apex.

Cerebral blood flow after craniotomy

Controlled ventilation induces hypocapnia; subsequently, it causes vasoconstriction and reduced cerebral blood flow. Monitor serum carbon dioxide levels routinely

Adventitious Breath Sounds

Crackles—discontinuous, explosive, bubbling sounds of short duration Cause: Air bubbling through fluid or mucus, or alveoli popping open on inspiration

Flail chest; *paradoxical chest movement.

Flail chest results from multiple rib fractures. This results in a "floating" section of ribs. Because this section is unattached to the rest of the bony rib cage, this segment results in paradoxical chest movement. This means that the force of inspiration pulls the fractured segment inward, while the rest of the chest expands. Similarly, during exhalation, the segment balloons outward while the rest of the chest moves inward. This is a characteristic sign of flail chest.

A clinic nurse notes that large numbers of clients present with flu-like symptoms. Which recommendations should the nurse include in the plan of care for these clients? Select all that apply.

Get plenty of rest. Increase intake of liquids. Take antipyretics for fever. Eat fruits and vegetables high in vitamin C.

The client is admitted to the hospital with a diagnosis of Legionnaires' disease. The nurse is providing information regarding the disease and treatment expectations. Which statement by the client indicates an understanding of the disease and treatments? **"I cannot give Legionnaires' disease to other people.

Legionnaires' disease is spread through infected aerosolized water. The mode of transmission is not person to person.

The nurse is assessing the respiratory status of a client who has suffered a fractured rib. The nurse should expect to note which finding? ** Pain, especially with inspiration

Rib fractures are a common injury, especially in the older client, and result from a blunt injury or a fall. Typical signs and symptoms include: pain and tenderness localized at the fracture site that is exacerbated by inspiration and palpation, shallow respirations, splinting or guarding the chest protectively to minimize chest movement, and possible bruising at the fracture site.

The nurse is taking the history of a client with occupational lung disease (silicosis). The nurse should assess whether the client wears which item during periods of exposure to silica particles? *MASK

Silicosis results from chronic, excessive inhalation of particles of free crystalline silica dust. The client should wear a mask to limit inhalation of this substance, which can cause restrictive lung disease after years of exposure.

The patient had video-assisted thoracic surgery (VATS) to perform a lobectomy. What does the nurse know is the reason for using this type of surgery?

The VATS procedure uses minimally invasive incisions that cause less discomfort and allow faster healing and return to normal activity as well as lower morbidity risk and fewer complications. Many surgeries can be done for lung cancer, but pneumonectomy via thoracotomy is the most common surgery for lung cancer. The incision for a thoracotomy is commonly a medial sternotomy or a lateral approach. A chest tube will be needed postoperatively for VATS.

A nurse is reading the report for a chest x-ray study in a client who has just been intubated. The report states that the tip of the endotracheal tube lies 1 cm above the carina. The nurse determines that the tube is positioned above which area of the respiratory system? **The bifurcation of the right and left main bronchi

The carina is a cartilaginous ridge that separates the openings of the two main (right and left) bronchi. If an endotracheal tube is inserted past the carina, the tube will enter the right main bronchus as a result of the natural curvature of the airway. This is hazardous because then only the right lung will be ventilated. Incorrect tube placement is easily detected because only the right lung will have breath sounds and rise and fall with ventilation.

A client with an endotracheal tube attached to mechanical ventilation begins to cough, and his face appears flushed. Which action should the nurse take first? **Suction the client through the endotracheal tube.

The client is choking on his secretions, which should be removed by suctioning of the endotracheal tube. There is no need at this time to contact the health care provider or call for respiratory therapy. The nurse should check the client's blood pressure, but suctioning is the priority.

Bronchoscopy **bronchospasm

The client should be assessed for signs/symptoms of complications, which would include cyanosis, dyspnea, stridor, bronchospasm, hemoptysis, hypotension, tachycardia, and dysrhythmias.

A nurse in an ambulatory clinic is preparing to administer a tuberculin skin test (Mantoux test) to a client who may have been exposed to a person with tuberculosis (TB). The client reports having received the bacille Calmette-Guérin (bCG) vaccine before moving to the United States from a foreign country. Which interpretation should the nurse make?

The client's test result will be positive, and a chest x-ray study will be required for evaluation.

A client with chronic obstructive pulmonary disease (COPD) is experiencing exacerbation of the disease. The nurse should determine that which finding documented in the client's record is an expected finding with this client? **Hyperinflation of lungs documented by chest x-ray

The clinical manifestations of COPD are several, including hypoxemia; hypercapnia; dyspnea on exertion and at rest; oxygen desaturation with exercise; use of accessory respiratory muscles; and prolonged exhalation. Chest x-ray results indicate a hyperinflated chest and may indicate a flattened diaphragm if the disease is advanced.

After Laryngectomy and a cuffed tracheostomy tube in place

The cuff should be inflated before the feeding to block the trachea and prevent food from entering.

A client is on continuous mechanical ventilation (CMV), and the low-pressure alarm sounds. The nurse should take which action? **Determine if there are any disconnections in the ventilator tubing.

The low pressure alarm can be caused by disconnected tubing, ETT cuff leak, or apnea. High-pressure alarms can be triggered by increased airway resistance, such as occurs with excess secretions in the airway, biting the tube, coughing, bronchospasm, a kinked ventilatory circuit, or excess condensation of water in the ventilator tubing.

The nurse is providing care for a client recently admitted with new onset pleurisy. Upon auscultation of the client's lungs, the nurse notes the absence of the pleural friction rub, which was documented on previous assessments. What is the most likely indication for this change in the client's lung sounds?

**Accumulation of pleural fluid in the inflamed area Pleural friction rub is auscultated early in the course of pleurisy before pleural fluid accumulates. Once fluid accumulates in the inflamed area, friction between the visceral and parietal lung surfaces decreases, and the pleural friction rub disappears.

The nurse is preparing to perform suctioning for a client with a tracheostomy tube and gathers the supplies needed for the procedure. What is the initial nursing action?

**Hyperoxygenate the client.

The nurse is preparing to give a bed bath to an immobilized client with tuberculosis. The nurse should wear which item when performing this care?

**Particulate respirator, gown, and gloves

The nurse is planning care for an 81-year-old unresponsive client admitted to the hospital with a medical diagnosis of pneumonia. The nurse has identified the problem of inability to clear the airway related to retained secretions. Which intervention is most appropriate?

**Provide nasotracheal suctioning as needed to remove secretions. Ineffective airway clearance reflects the client's inability to expectorate secretions. The intervention specifically addressing retained secretions is in the correct option.

The nurse should provide which home care instructions to a client who had a laryngectomy and has a stoma? Select all that apply. Increase the humidity in the home. Obtain and wear a Medic-Alert bracelet. Stay away from people who have a respiratory infection. Be careful with showering to avoid water from entering the stoma.

**Recalling that most interventions focus on protection of the stoma and the prevention of infection. The nurse should teach the client how to care for the stoma, depending on the type of laryngectomy performed. Most interventions focus on protection of the stoma and the prevention of infection. Interventions include obtaining a Medic-Alert bracelet, avoiding exposure to people with infections, avoiding swimming, using care when showering, and preventing debris from entering the stoma. Additional interventions include wearing a stoma guard or high-collared clothing to cover the stoma, increasing the humidity in the home, and increasing fluid intake to 3000 mL/day to keep the secretions thin.

Before discharge, the nurse discusses activity levels with a 61-year-old patient with chronic obstructive pulmonary disease (COPD) and pneumonia. Which exercise goal is most appropriate once the patient is fully recovered from this episode of illness?

**Walk for 20 min/day, keeping the pulse rate less than 130 beats/min. The patient will benefit from mild aerobic exercise that does not stress the cardiorespiratory system. The patient should be encouraged to walk for 20 min/day, keeping the pulse rate less than 75% to 80% of maximum heart rate (220 - patient's age).

The nurse is preparing to wean a client from a ventilator by the use of a T-piece. Which would be a component of the plan of care with this type of weaning process? Select all that apply

*Pressure support is added to the oxygen system. *The T-piece is connected to the client's artificial airway. *The client is removed from the mechanical ventilator for a short period of time. *Supplemental oxygen is provided through the T-piece at a fraction of inspired oxygen that is 10% higher than a ventilator setting.

The nurse reads a client's tuberculin (Mantoux) skin test as positive and notes that previous tests were negative. The client becomes upset and asks the nurse what this means. The nurse should base the response on which interpretation? **Exposure to tuberculosis

A client who tests positive on a tuberculin (Mantoux) skin test either has been exposed to tuberculosis or has inactive (dormant) tuberculosis. The client must then undergo chest radiography and sputum culture to confirm the diagnosis.

The nurse should check the results of which diagnostic test that will confirm the diagnosis of TB? *Sputum culture

A culture and isolation of Mycobacterium tuberculosis. A presumptive diagnosis is made based on a tuberculin skin test, a sputum smear that is positive for acid-fast bacteria, a chest x-ray, and histological evidence of granulomatous disease on biopsy.

The nurse is caring for a client with tuberculosis who is fearful of the disease and anxious about the prognosis. In planning nursing care, the nurse should incorporate which intervention as the best strategy to assist the client in coping with the illness? **Provide reassurance that continued compliance with medication therapy is the most proactive way to cope with the disease.

A primary role of the nurse working with a client with tuberculosis is to teach the client about medication therapy. An anxious client might not absorb information optimally. The nurse continues to reinforce teaching using a variety of methods (repetition, teaching aids) and teaches the family about the medications as well. The most effective way of coping with the disease is to learn about the therapy that will eradicate it. This gives the client a measure of power over the situation and outcome.

A nurse is planning care for a client who is scheduled for a tracheostomy procedure. What equipment should the nurse plan to have at the bedside when the client returns from surgery? **Obturator

A replacement tube of the same size and an obturator are kept at the bedside at all times in case the tracheostomy tube becomes dislodged. Additionally, a curved hemostat that could be used to hold the trachea open if dislodgement occurs should also be kept at the bedside. An oral airway and epinephrine would not be needed.

The nurse determines that the patient understood medication instructions about the use of a spacer device when taking inhaled medications after hearing the patient state what as the primary benefit? ** "More of the medication will get down into my lungs to help my breathing."

A spacer assists more medication to reach the lungs, with less being deposited in the mouth and the back of the throat. It does not affect the cost or increase the speed of using the inhaler.

Which clinical manifestation should the nurse expect to find during assessment of a patient admitted with pneumonia? * increased vocal fremitus on palpation.

A typical physical examination finding for a patient with pneumonia is increased vocal fremitus on palpation. Other signs of pulmonary consolidation include bronchial breath sounds, egophony, and crackles in the affected area. With pleural effusion, there may be dullness to percussion over the affected area.

A 74-year-old male client is admitted to the intensive care unit (ICU) with a diagnosis of respiratory failure secondary to pneumonia. Currently, he is ventilator-dependent, with settings of tidal volume (VT) of 750 mL and an intermittent mandatory ventilation (IMV) rate of 10 breaths/min. Arterial blood gas (ABG) results are as follows: pH, 7.48; Paco2, 30 mm Hg; Pao2, 64 mm Hg; HCO3, 25 mEq/L; and Fio2, 0.80. Which intervention should the nurse implement first?

Add 5 cm positive end-expiratory pressure (PEEP). Adding PEEP (D) helps improve oxygenation while reducing Fio2 to a less toxic level.

A client with long-standing empyema undergoes decortication of the affected lung area. Postoperatively the nurse should place the client in which position? **Semi-Fowler's

After any procedure involving lung surgery, the nurse should position the client in semi-Fowler's position. This position allows for maximal lung expansion and promotes drainage through chest tubes that may be placed during surgery. The positions identified in the remaining options will limit lung expansion.

The nurse is providing immediate postprocedure care to a client who had a thoracentesis to relieve a tension pneumothorax that resulted from rib fractures. The goal is that the client will exhibit normal respiratory functioning, and the nurse provides instructions to assist the client toward this goal. Which client statement indicates that further instruction is needed? **"I will lie on the affected side for an hour."

After the procedure the client usually is turned onto the unaffected side for 1 hour to facilitate lung expansion. Tachypnea, dyspnea, cyanosis, retractions, or diminished breath sounds, which may indicate pneumothorax, should be reported to the health care provider. A chest x-ray may be performed to evaluate the degree of lung re-expansion or pneumothorax. Subcutaneous emphysema (crepitus) may follow this procedure because air in the pleural cavity leaks into subcutaneous tissues. The involved tissues feel like lumpy paper and crackle when palpated (crepitus). Usually subcutaneous emphysema causes no problems unless it is increasing and constricting vital organs, such as the trachea.

The nurse has completed care for a client whose tracheostomy tube has a nondisposable inner cannula. Which action should the nurse perform prior to reinserting the inner cannula? **Allow the inner cannula to dry after washing it with sterile water.

After washing and rinsing the inner cannula with sterile water (per agency policy), the nurse taps it against a sterile surface to remove excess liquid and allows it to dry. The nurse then inserts the cannula into the tracheostomy tube and turns it clockwise to lock it into place.

The nurse evaluates that nursing interventions to promote airway clearance in a patient admitted with COPD are successful based on which finding? ** Effective and productive coughing

Airway clearance is most directly evaluated as successful if the patient can engage in effective and productive coughing. Absence of dyspnea, improved mental status, and PaO2 within normal range for the patient show improved respiratory status but do not evaluate airway clearance.

The patient has an order for each of the following inhalers. Which one should the nurse offer to the patient at the onset of an asthma attack? **Albuterol (Proventil)

Albuterol is a short-acting bronchodilator that should be given initially when the patient experiences an asthma attack. Salmeterol (Serevent) is a long-acting β2-adrenergic agonist, which is not used for acute asthma attacks. Beclomethasone (Qvar) is a corticosteroid inhaler and not recommended for an acute asthma attack. Ipratropium bromide (Atrovent) is an anticholinergic agent that is less effective than β2-adrenergic agonists. It may be used in an emergency with a patient unable to tolerate short-acting β2-adrenergic agonists (SABAs)

When admitting a patient with a diagnosis of asthma exacerbation, the nurse will assess for what potential triggers (select all that apply)? Exercise Allergies Emotional stress Upper respiratory infections

Although the exact mechanism of asthma is unknown, there are several triggers that may precipitate an attack. These include allergens, exercise, air pollutants, upper respiratory infections, drug and food additives, psychologic factors, and gastroesophageal reflux disease (GERD).

The nurse monitors the respiratory status of the client being treated for acute exacerbation of chronic obstructive pulmonary disease (COPD). Which assessment finding would indicate deterioration in ventilation? **Rapid, shallow respirations

An increase in the rate of respirations and a decrease in the depth of respirations together indicate deterioration in ventilation. Cyanosis is not a good indicator of oxygenation in the client with COPD. Cyanosis may be present with some but not all clients. A hyperinflated chest (barrel chest) and hypertrophy of the accessory muscles of the upper chest and neck are common features of chronic COPD. During an exacerbation, coarse crackles are expected to be heard bilaterally throughout the lungs but do not indicate deterioration in ventilation.

When admitting a 45-year-old female with a diagnosis of pulmonary embolism, the nurse will assess the patient for which risk factors (select all that apply)? Obesity Malignancy Cigarette smoking Prolonged air travel

An increased risk of pulmonary embolism is associated with obesity, malignancy, heavy cigarette smoking, and prolonged air travel with reduced mobility. Other risk factors include deep vein thrombosis, immobilization, surgery within the last 3 months, oral contraceptives and hormone therapy, heart failure, pregnancy, and clotting disorders.

A nurse is providing instructions to a client about diaphragmatic breathing. The nurse tells the client that this technique is helpful because, in normal respiration, as the diaphragm contracts, it takes which action? **Moves downward and out

As the diaphragm contracts, it moves downward and out, becoming flatter and expanding the thoracic cage, to promote lung expansion. This process occurs during the inspiratory phase of the respiratory cycle. The incorrect options occur with exhalation and relaxation of the diaphragm.

The nurse is assisting a patient to learn self-administration of beclomethasone, two puffs inhaled every 6 hours. What should the nurse explain as the best way to prevent oral infection while taking this medication? **Rinse the mouth with water following the second puff of medication.

Because beclamethosone is a corticosteroid, the patient should rinse the mouth with water following the second puff of medication to reduce the risk of fungal overgrowth and oral infection.

The nurse is performing nasotracheal suctioning of a client. The nurse determines that the client is adequately tolerating the procedure if which observation is made? **Coughing occurs with suctioning.

Because the cough reflex normally is present and suctioning triggers coughing, this is the preferable option from the two remaining. The nurse monitors for adverse effects of suctioning: Cyanosis, excessively rapid or slow heart rate, and sudden development of bloody secretions. If any of these signs is observed, the nurse immediately stops suctioning and reports the adverse effect to the health care provider. Coughing is a normal response to suctioning for the client with an intact cough reflex and does not indicate that he or she cannot tolerate the procedure.

A 71-year-old patient is admitted with acute respiratory distress related to cor pulmonale. Which nursing intervention is most appropriate during admission of this patient?

Because the patient is having respiratory difficulty, the nurse should ask specific questions about this episode and perform a physical assessment of this system. Further history taking and physical examination of other body systems can proceed once the patient's acute respiratory distress is being managed.

When caring for a patient with chronic obstructive pulmonary disease (COPD), the nurse identifies a nursing diagnosis of imbalanced nutrition: less than body requirements after noting a weight loss of 30 lb. Which intervention should the nurse add to the plan of care for this patient? ** Order a high-calorie, high-protein diet with six small meals a day.

Because the patient with COPD needs to use greater energy to breathe, there is often decreased oral intake because of dyspnea. A full stomach also impairs the ability of the diaphragm to descend during inspiration, thus interfering with the work of breathing. For these reasons, the patient with COPD should eat six small meals per day taking in a high-calorie, high-protein diet, with non-protein calories divided evenly between fat and carbohydrate. The other interventions will not increase the patient's caloric intake.

The nurse is caring for a client who recently returned from another country who exhibits signs and symptoms suspicious of severe acute respiratory syndrome (SARS). Which clinical manifestations support this diagnosis? Select all that apply. ** Dry cough - Shortness of breath and Fever greater than 100.4.

Between two and seven days after the onset of SARS, which is caused by a coronavirus, clients exhibit a dry cough. SARS is an acute viral respiratory infection that results in respiratory signs and symptoms, including difficulty breathing and shortness of breath. SARS, a viral infection, generally begins with a fever greater than 100.4º F, headache, and muscle weakness. Although clients may exhibit sinus tachycardia, chest pains are not a typical symptom associated with SARS. The cough associated with SARS is nonproductive and hemoptysis does not occur.

The patient who had idiopathic pulmonary fibrosis had a bilateral lung transplantation. Now he is experiencing airflow obstruction that is progressing over time. It started with a gradual onset of exertional dyspnea, nonproductive cough, and wheezing. What are these manifestations signs of in the lung transplant patient? **Bronchiolitis obliterans (BOS)

Bronchiolitis obliterans (BOS) is a manifestation of chronic rejection and is characterized by airflow obstruction progressing over time with a gradual onset of exertional dyspnea, nonproductive cough, wheezing, and/or low-grade fever. Pulmonary infarction occurs with lack of blood flow to the bronchial tissue or preexisting lung disease. With pulmonary hypertension, the pulmonary pressures are elevated and can be idiopathic or secondarily due to parenchymal lung disease that causes anatomic or vascular changes leading to pulmonary hypertension. CMV pneumonia is the most common opportunistic infection 1 to 4 months after lung transplant.

A nurse is monitoring the status of a client who is being treated for dyspnea. The nurse is aware that which factor will decrease the work of breathing for this client? **Bronchodilation

Bronchodilation decreases airway resistance and decreases the work of breathing for the client with dyspnea. Clients with dyspnea who have bronchospasm, increased mucus production, or edema exhibit increased airway resistance, which increases the work of breathing.

The nurse is monitoring the chest tube drainage system in a client with a chest tube. The nurse notes intermittent bubbling in the water seal chamber. Which is the most appropriate nursing action? *Document the findings.

Bubbling in the water seal chamber is caused by air passing out of the pleural space into the fluid in the chamber. Intermittent (not constant) bubbling is normal. It indicates that the system is accomplishing one of its purposes, removing air from the pleural space. Continuous bubbling during inspiration and expiration indicates that an air leak exists. If this occurs, it must be corrected. Notifying the health care provider and changing the chest tube drainage system are not indicated at this time.

A client with chronic obstructive pulmonary disease (COPD) has a respiratory rate of 24 breaths per minute, bilateral crackles, and cyanosis and is coughing but is unable to expectorate sputum. Which problem is the priority? **Inability to clear the airway related to inability to expectorate sputum

COPD is a term that represents the pathology and symptoms that occur with clients experiencing both emphysema and chronic bronchitis. All the problems listed are potentially appropriate for a client with COPD. For the nurse prioritizing this client's problems, it is important first to maintain circulation, airway, and breathing. At present, the client demonstrates problems with ventilation because of ineffective coughing, so the correct option would be the priority problem. The bilateral crackles would suggest fluid or sputum in the alveoli or airways; however, the client is unable to expectorate this sputum. The client's respiratory rate is only slightly elevated, so option 3, altered breathing pattern is not as important as airway. The client is cyanotic, but this probably is owing to the ineffective clearance of the sputum, causing poor gas exchange. The data in the question do not support low cardiac output as being most important at this time.

After a bronchoscopy because of suspected cancer of the lung, a client develops pleural effusion. What should the nurse conclude is the most likely cause of the pleural effusion?

Cancerous lesions in the pleural space increase the osmotic pressure, causing a shift of fluid to that space. Excessive fluid intake is usually balanced by increased urine output. Inadequate chest expansion results from pleural effusion and is not the cause of it. A bronchoscopy does not involve the pleural space.

Carbon Monoxide Poisoning **flushing,

Carbon monoxide levels between 11% and 20% result in flushing, headache, decreased visual activity, decreased cerebral functioning, and slight breathlessness; levels of 21% to 40% result in nausea, vomiting, dizziness, tinnitus, vertigo, confusion, drowsiness, pale to reddish-purple skin, tachycardia; levels of 41% to 60% result in seizure and coma; and levels higher than 60% result in death.

A client with a history of recent upper respiratory infection comes to the urgent care center complaining of chest pain. The nurse determines that the pain is most likely of a respiratory origin if the client makes which statement about the pain? **"It hurts more when I breathe in."

Chest pain is assessed by using the standard pain assessment parameters, such as characteristics, location, intensity, duration, precipitating and alleviating factors, and associated symptoms. Pain of pleuropulmonary (respiratory) origin usually worsens on inspiration

An ambulatory care nurse is assessing a client with chronic sinusitis. The nurse would expect to note which assessment findings in this client? Select all that apply. Anosmia Chronic cough Purulent nasal discharge

Chronic sinusitis is characterized by persistent purulent nasal discharge, a chronic cough due to nasal discharge, anosmia (loss of smell), nasal stuffiness, and headache that is worse on arising after sleep.

The nurse is caring for a client who is on strict bed rest and develops a plan of care with goals related to the prevention of deep vein thrombosis and pulmonary emboli. Which nursing action is most helpful in preventing these disorders from developing? ** Encouraging active range-of-motion exercises

Clients at greatest risk for deep vein thrombosis and pulmonary emboli are immobilized clients. Basic preventive measures include early ambulation, leg elevation, active leg exercises, elastic stockings, and intermittent pneumatic calf compression. Keeping the client well hydrated is essential because dehydration predisposes to clotting. A pillow under the knees may cause venous stasis. Heat should not be applied without a health care provider's prescription.

A nurse working on a medical respiratory nursing unit is caring for several clients with respiratory disorders. The nurse should determine that which client on the nursing unit is at the lowest risk for infection with tuberculosis? **A man who is an inspector for the US Postal Service

Clients at high risk for acquiring tuberculosis include immigrants from Asia, Africa, Latin America, and Oceania; medically underserved populations (ethnic minorities, homeless); those with human immunodeficiency virus infection or other immunosuppressive disorders; residents in group settings (long-term care, correctional facilities); and health care workers.

The nurse is caring for a client hospitalized with acute exacerbation of chronic obstructive pulmonary disease. Which finding would the nurse expect to note on assessment of this client? Select all that apply. A hyperinflated chest noted on the chest x-ray Decreased oxygen saturation with mild exercise

Clinical manifestations of chronic obstructive pulmonary disease (COPD) include hypoxemia, hypercapnia, dyspnea on exertion and at rest, oxygen desaturation with exercise, and the use of accessory muscles of respiration. Chest x-rays reveal a hyperinflated chest and a flattened diaphragm if the disease is advanced. Pulmonary function tests will demonstrate decreased vital capacity.

The nurse caring for a client who is mechanically ventilated is monitoring for complications of mechanical ventilation. Which assessment finding, if noted by the nurse, indicates the need for follow-up? **A blood pressure of 90/60 mm Hg decreased from 112/78 mm Hg

Complications of mechanical ventilation include the following: (1) hypotension caused by application of positive pressure, which increases intrathoracic pressure and inhibits blood return to the heart; (2) pneumothorax or subcutaneous emphysema as a result of positive pressure; (3) gastrointestinal alterations such as stress ulcers; (4) malnutrition if nutrition is not maintained; (5) infections; (6) muscular deconditioning; and (7) ventilator-dependence or inability to wean. Some muscle weakness is expected.

When assessing a client with pleural effusion, the nurse expects to identify: **Reduced or absent breath sounds at the base of the lungs.

Compression of the lung by fluid that accumulates at the base of the lungs reduces lung expansion and air exchange. There is no fluid in the alveoli, so no cracklesare produced. If there is tracheal deviation, it is away from the involved side. Dullness is produced on percussion of the involved area.

The nurse determines the client with a chest tube to a closed drainage system is experiencing an air leak. Which finding is indicative of this? **Continuous bubbling is observed in the water seal during inspiration and expiration.

Continuous bubbling in the water seal chamber during inspiration and expiration indicates that air is leaking into the drainage system or pleural cavity. Bubbling is an expected finding in the suction control chamber when the device is connected to suction. Tidaling is a normal phenomenon. Absence of tidaling can be indicative of re-expansion of the lung or obstruction or kinking of the chest tube.

Nursing assessment findings of jugular venous distention and pedal edema would be indicative of what complication of chronic obstructive pulmonary disease (COPD)? ** Fluid volume excess resulting from cor pulmonale

Cor pulmonale is a right-sided heart failure caused by resistance to right ventricular outflow resulting from lung disease. With failure of the right ventricle, the blood emptying into the right atrium and ventricle would be slowed, leading to jugular venous distention and pedal edema.

A nurse is caring for the client who is suspected of having lung cancer. The nurse should assess the client for which most frequent early symptom of lung cancer? **Cough

Cough is the most frequent early symptom of lung cancer, which begins as nonproductive and hacking, and progresses to productive. In the smoker who already has a cough, a change in the character and frequency of cough usually occurs. Hoarseness indicates that the affected tissue is in the upper airway. Wheezing and blood-streaked sputum (hemoptysis) are later signs of lung cancer. Pain is a very late sign and is usually pleuritic in nature.

Crackles: CLINICAL SIGNIFICANCE: Atelectasis, fluid retention in small airways (pulmonary edema), retention of mucus (bronchitis, pneumonia), interstitial fibrosis

Crackles: Fine: soft, short duration Coarse: loud, longer duration Wet or dry May disappear after coughing, suctioning, or deep inspiration if alveoli remain inflated

The nurse is assessing the functioning of a chest tube drainage system in a client who has just returned from the recovery room following a thoracotomy with wedge resection. Which are the expected assessment findings? Select all that apply.

Drainage system maintained below the client's chest 50 mL of drainage in the drainage collection chamber Occlusive dressing in place over the chest tube insertion site Fluctuation of water in the tube in the water seal chamber during inhalation and exhalation

The nurse is giving discharge instructions to a client with pulmonary sarcoidosis. The nurse concludes that the client understands the information if the client indicates to report which early sign of exacerbation? **Shortness of breath

Dry cough and dyspnea are typical early manifestations of pulmonary sarcoidosis. Later manifestations include night sweats, fever, weight loss, and skin nodules.

Monitor for signs of pulmonary embolus: Dyspnea Hemoptysis Impeding Doom Apprehension/Hyperalert Crackles

Dyspnea is the most common symptom of a pulmonary embolus because of increased alveolar dead space, which impedes ventilation. With a pulmonary embolus, pulmonary blood flow is obstructed partially or completely; when infarcted areas have alveolar damage, red blood cells move into alveoli, resulting in hemoptysis. Clients with a pulmonary embolus have severe dyspnea and chest pain that precipitate a feeling of impending doom. Clients with a pulmonary embolus usually are apprehensive and hyperalert, not apathetic. Crackles, not bronchial wheezes, occur.

The nurse is developing a plan of care for a client at risk for acute respiratory distress syndrome (ARDS). As part of the plan, the nurse will check for which item to detect an early sign of this disorder? **DYSpnea

In most cases of ARDS, tachypnea and dyspnea are the first clinical manifestations. Blood-tinged frothy sputum would present as a later sign, after the development of pulmonary edema. Breath sounds in the early stages of ARDS usually are clear.

A nurse is caring for several postoperative clients. For what clinical manifestations of a pulmonary embolus should the nurse monitor these clients? Select all that apply.

Dyspnea is the most common symptom of a pulmonary embolus because of increased alveolar dead space, which impedes ventilation. With a pulmonary embolus, pulmonary blood flow is obstructed partially or completely; when infarcted areas have alveolar damage, red blood cells move into alveoli, resulting in *hemoptysis. Clients with a pulmonary embolus have severe dyspnea and *chest pain that precipitate a feeling of *impending doom. Clients with a pulmonary embolus usually are apprehensive and * hyperalert, not apathetic. Crackles, not bronchial wheezes, occur. Wheezes are associated with reactive airway disorders, such as asthma.

A client is on mechanical ventilation. When condensation collects in the ventilator tubing, the nurse should:

Emptying the fluid from the tubing is necessary to prevent flooding of the trachea with fluid; some systems have receptacles attached to the tubing to collect the fluid and others have to be temporarily disconnected while emptying the fluid. This circumstance does not require assistance from a respiratory therapist. Humidity is necessary to preserve moistness of the respiratory tract and help liquefy secretions. The amount of condensation is irrelevant when recording total intake and output.

Herpes Education

Encourage annual physical and Pap smear. Take antiviral medication as prescribed. Use condoms to avoid transmission to others. Warm sitz baths may relieve itching.

Which nursing interventions are appropriate in caring for a client with emphysema? Select all that apply. *Teach diaphragmatic and pursed-lip breathing. *Encourage alternating activity with rest periods. *Teach the client techniques of chest physiotherapy.

Fluids are encouraged, not reduced, to liquefy secretions for easier expectoration. Diaphragmatic and pursed-lip breathing assists in opening alveoli and ease dyspnea. The client should be encouraged to perform activities and exercise as tolerated, such as dressing and walking, with rest periods in between. Chest physiotherapy consists of percussion, vibration, and postural drainage. These techniques are helpful in removing secretions. Elevating the head of the bed assists with breathing.

The clinic nurse is providing instructions to a client with a diagnosis of pharyngitis. The nurse provides which instruction to the client? **Avoid foods that are highly seasoned.

Foods that are highly seasoned are irritating to the throat and should be avoided. The client with pharyngitis should be instructed to consume cool clear fluids, ice chips, or ice pops to soothe the painful throat. Citrus products should be avoided because they irritate the throat. Milk and milk products are avoided because they tend to increase mucus production. The client should be instructed to eat bland foods and drink 2000 to 3000 mL of fluid daily unless contraindicated.

A nurse is teaching a client with pulmonary disease about fundamental concepts of gas exchange. When asked for further details by the client, the nurse explains that gas exchange occurs through which process? **Diffusion

Gas exchange occurs by diffusion, which means that oxygen and carbon dioxide move across the alveolar-capillary membrane as a result of a pressure gradient. Osmosis is the process of movement according to a concentration gradient. Ionization refers to the process whereby a molecule gains or loses electrons. Active transport is movement of molecules by carrying them across a cell membrane.

Restrictive airway disease - Early signs of respiratory acidosis

Headache is a symptom of cerebral hypoxia associated with early respiratory acidosis. Irritability is a sign of cerebral hypoxia associated with early respiratory acidosis. Restlessness is a sign of cerebral hypoxia associated with early respiratory acidosis. Lightheadedness is a symptom of respiratory alkalosis, not acidosis.

A client is on continuous mechanical ventilation (CMV) and the high-pressure alarm sounds. Which action should the nurse take to eliminate the problem? ** Empty excess accumulated water from the ventilatory circuit tubing.

High-pressure alarms can be triggered by increased airway resistance caused by excess secretions in the airway, biting the tube, coughing, bronchospasm, a kinked ventilatory circuit, or excess condensation of water in the ventilator tubing. Excess water should be emptied from the tubing. Alarms should never be silenced until the cause has been identified and corrected. Additionally, this will not eliminate the problem. The low-pressure alarm would sound with a disconnection. Filling the cuff to 25 mm Hg can result in impaired circulation to the tracheal mucosa.

The nurse is providing an educational session to community members regarding histoplasmosis. The nurse should provide which information about this disease? **It can be caused by the inhalation of spores from bird droppings.

Histoplasmosis is caused by the inhalation of spores from bat or bird droppings. This disease cannot be transmitted from one person to another. Lyme disease is a multisystem infection that results from a bite by a tick that is usually carried by several species of deer. Toxoplasmosis is caused by the ingestion of cysts from contaminated cat feces.

Asthma & Surgery There will be an increased risk of respiratory tract infections.

Hypersecretion of the mucous glands provides an excellent warm, moist medium for microorganisms. Asthma is not a disease that is voluntarily controlled. Coughing must be encouraged; it prevents retention of mucus, which is an excellent medium for microorganisms. Excessive secretions also limit gaseous exchange. The anesthesiologist will make recommendations about the type of anesthesia best suited for the client and the surgical procedure.

The nurse is monitoring the respiratory status of a client after creation of a tracheostomy. The nurse understands that oxygen saturation measurements obtained by pulse oximetry may be inaccurate if the client has which coexisting problem? **Hypotension

Hypotension, shock, or the use of peripheral vasoconstricting medications may result in inaccurate pulse oximetry readings as a result of impaired peripheral perfusion. Pulse oximetry measures oxygen saturation in blood flowing through the blood vessels in the periphery of the body and that inaccurate measurement may result from any factor that impairs blood flow through the periphery.

A health care provider writes a prescription to begin to wean the client from the mechanical ventilator by use of intermittent mandatory ventilation/synchronized intermittent mandatory ventilation (IMV/SIMV). The nurse determines that the process of weaning will occur by which mechanism? ***Gradually decreasing the respiratory rate until the client can assume the work of breathing without ventilatory assistance.

IMV/SIMV is one of the methods used for weaning. With this method the respiratory rate is gradually decreased until the client assumes all of the work of breathing on his or her own. This method works exceptionally well in the weaning of clients from short-term mechanical ventilation, such as that used in clients who have undergone surgery. The respiratory rate frequently is decreased in increments on an hourly basis until the client is weaned and is ready for extubation.

A nurse is providing morning care to a client who has a closed chest tube drainage system to treat a pneumothorax. When the nurse turns the client to the side, the chest tube is accidentally dislodged from the chest. The nurse immediately applies sterile gauze over the chest tube insertion site. Which is the nurse's next action? ** Cal the HCP

If the chest drainage system is dislodged from the insertion site, the nurse immediately applies sterile gauze over the site and calls the health care provider. The nurse would maintain the client in an upright position. A new chest tube system may be attached if the tube requires insertion, but this would not be the next action.

A registered nurse who is orienting a new nursing graduate to the hospital emergency department instructs the new graduate to monitor a client for one-sided chest movement on the right side while the client is being intubated by the health care provider. This instruction is based on the understanding that the endotracheal tube could enter which respiratory structures? **Right main bronchus if inserted too far

If the endotracheal tube is inserted too far into the client's trachea, the tube will enter the right main bronchus. This occurs because the right bronchus is shorter and wider than the left and extends downward in a more vertical plane. If the tube is not inserted far enough, no chest expansion at all will occur.

The nurse assesses for one-sided chest movement on the right while a client is being intubated by the health care provider. The nurse's action is based on the possibility that which could occur with the endotracheal tube? ***It could enter the right main bronchus if inserted too far.

If the endotracheal tube is inserted too far, the tube will travel past the trachea and enter the right main bronchus. This occurs because the right bronchus is shorter and wider than the left and extends downward in a more vertical plane.

The nurse is caring for a client on a mechanical ventilator. The high-pressure alarm sounds. The nurse assesses the client and attempts to determine the cause of the alarm. Which initial nursing action would be appropriate if the nurse is unable to determine the cause of ventilator alarm? ***Disconnect the client from the ventilator and manually ventilate the client with a resuscitation device.

If the nurse is unable to troubleshoot an alarm or suspects equipment failure in a mechanical ventilator, the nurse should manually ventilate the client with a resuscitation device. The nurse should never shut off the alarm. It is not necessary to contact the HCP, although the respiratory therapist may be notified to assist in troubleshooting the cause of the problem. However, the initial nursing action would be to manually ventilate the client.

While changing the tapes on a tracheostomy tube, the client coughs and the tube is dislodged. Which is the initial nursing action? **Grasp the retention sutures to spread the opening.

If the tube is dislodged accidentally, the initial nursing action is to grasp the retention sutures and spread the opening. If agency policy permits, the nurse then attempts immediately to replace the tube. Calling ancillary services or the HCP will delay treatment in this emergency situation.

A client begins to experience drainage of small amounts of bright red blood from the tracheostomy tube 24 hours after a supraglottic laryngectomy. Which is the best nursing action? **Notify the health care provider (HCP).

Immediately after laryngectomy, a small amount of bleeding occurs from the tracheostomy that resolves within the first few hours. Otherwise, bleeding that is bright red may be a sign of impending rupture of a vessel. The bleeding in this instance represents a potential threat to life, and the HCP is notified to further evaluate the client and suture or repair the source of the bleeding. The other options do not address the urgency of the problem. Failure to notify the HCP places the client at risk.

A nurse is caring for a client who has just returned from the postanesthesia care unit after radical neck dissection. The nurse should assess for which characteristic of wound drainage expected in the immediate postoperative period? **serosanguineous fluid.

Immediately after radical neck dissection, the client will have a wound drain in the neck attached to portable suction, which drains

When planning teaching for the patient with chronic obstructive pulmonary disease (COPD), the nurse understands that what causes the manifestations of the disease? **Hyperinflation of alveoli and destruction of alveolar walls

In COPD there are structural changes that include hyperinflation of alveoli, destruction of alveolar walls, destruction of alveolar capillary walls, narrowing of small airways, and loss of lung elasticity. An autosomal recessive deficiency of antitrypsin may cause COPD. Not all patients with COPD have excess mucus production by the increased number of goblet cells.

The client is returned to the nursing unit following thoracic surgery with chest tubes in place. During the first few hours postoperatively, the nurse assesses for drainage and expects to note which characteristics? **The drainage is bloody.

In the first few hours after surgery the drainage from the chest tube is bloody. After several hours it becomes serosanguineous. The client should not experience frequent clotting. Proper chest tube function should allow for drainage of blood before it has the chance to clot in the chest or the tubing.

A client is returned to the nursing unit after thoracic surgery with chest tubes in place. During the first few hours postoperatively, what type of drainage should the nurse expect? *Bloody

In the first few hours after surgery, the drainage from the chest tube is bloody. After several hours, it becomes serosanguineous. The client should not experience frequent clotting. Proper chest tube function should allow for drainage of blood before it has the chance to clot in the chest or the tubing.

client with uncomplicated or simple silicosis is being monitored yearly at the health care clinic. In this type of silicosis, which symptom should the nurse expect that the client will exhibit? **No symptoms

In uncomplicated or simple silicosis, the client would be asymptomatic, although evidence of fibrosis on an x-ray would be present. Malaise, extreme fatigue, anorexia, weight loss, and severe dyspnea on exertion would occur in a client with chronic complicated silicosis.

TB - Tuberculin skin test

Induration measuring 10 mm or more is considered to be a positive result in children younger than 4 years of age and in children with chronic illness or at high risk for exposure to tuberculosis. A reaction of 5 mm or more is considered to be a positive result for the highest risk groups, such as a child with an immunosuppressive condition or a child with human immunodeficiency virus (HIV) infection. A reaction of 15 mm or more is positive in children 4 years or older without any risk factors.

The nurse provides instructions about how to use a nebulizer to a client with chronic obstructive pulmonary disease (COPD). The nurse concludes that additional teaching is needed when the client: **Inhales with the lips tightly sealed around the mouthpiece of the nebulizer.

Inhaling with the lips tightly sealed around the mouthpiece of the nebulizer results in nasal breathing, which negates the effects of aerosol medication. The mouthpiece should be gently held in the mouth just past the lips. The nebulizer tip should be past the lips to deliver the medication. Holding the inspired breath for at least three seconds promotes contact of the medication with the bronchial mucosa. Exhaling slowly through the mouth with lips pursed slightly prolongs and improves delivery of the medication to the respiratory mucosa.

Several days after a client had a total laryngectomy, the health care provider prescribes a progressive diet as tolerated. What should the nurse do? ** Keep suction apparatus readily available in case excessive respiratory secretions occur

Initial attempts at oral feeding may cause a choking feeling that may produce severe coughing that raises secretions. Effective coughing is difficult because with a laryngectomy there is no glottis to close to assist with an expulsive cough. Excessive secretions may block the airway and must be removed by suctioning. Swallowing does not have an adverse effect on the suture line; a nasogastric tube is not used because it can traumatize the suture line. A progressive diet is started with liquids, not pureed foods. Airway patency is the priority, not administering medication for pain.

The nurse is discussing the techniques of chest physiotherapy and postural drainage (respiratory treatments) to a client having expectoration problems because of chronic thick, tenacious mucus production in the lower airway. The nurse explains that after the client is positioned for postural drainage the nurse will perform which action to help loosen secretions? **Percussion and vibration.

It helps loosen secretions in the smaller lower airways. Postural drainage positions the client so that gravity can help mucus moving from smaller airways to larger ones to support expectoration of the mucus.

The nurse is teaching a patient how to self-administer ipratropium (Atrovent) via a metered dose inhaler (MDI). Which instruction given by the nurse is most appropriate to help the patient learn the proper inhalation technique? ** "Breathe out slowly before positioning the inhaler."

It is important to breathe out slowly before positioning the inhaler. This allows the patient to take a deeper breath while inhaling the medication, thus enhancing the effectiveness of the dose. The inhaler should be shaken well. A spacer may be used. Holding the breath after the inhalation of medication helps keep the medication in the lungs, but 30 seconds will not be possible for a patient with COPD.

A nursing student is developing a plan of care for a client with a chest tube that is attached to a Pleur-Evac drainage system. Which intervention in the care plan indicates the need for further teaching for the student? **Instruct the client to avoid coughing and deep breathing.

It is important to encourage the client to cough and deep breathe when a chest tube drainage system is in place. This will assist in facilitating appropriate lung re-expansion. The client is positioned in semi-Fowler's position to facilitate ease in breathing. Water is added to the suction chamber as it evaporates to maintain the full suction level prescribed. Connections between the chest tube and the drainage system are taped to prevent accidental disconnection.

The nurse is evaluating if a patient understands how to safely determine whether a metered dose inhaler (MDI) is empty. The nurse interprets that the patient understands this important information to prevent medication underdosing when the patient describes which method to check the inhaler? ** Keep track of the number of inhalations used.

It is no longer appropriate to see if a canister floats in water or not since this is not an accurate way to determine the remaining inhaler doses. The best method to determine when to replace an inhaler is by knowing the maximum puffs available per MDI and then replacing it after the number of days when those inhalations have been used. (100 puffs/2 puffs each day = 50 days)

A patient has been receiving oxygen per nasal cannula while hospitalized for COPD. The patient asks the nurse whether oxygen use will be needed at home. What is the most appropriate response by the nurse? **You will not need oxygen until your oxygen saturation drops to 88% and you have symptoms of hypoxia."

Long-term oxygen therapy in the home will not be considered until the oxygen saturation is less than or equal to 88% and the patient has signs of tissue hypoxia, such as cor pulmonale, erythrocytosis, or impaired mental status. PaO2 less than 55 mm Hg will also allow home oxygen therapy to be considered.

A client reports having a bad cold and chest pain that worsens when the client takes deep breaths. Where should the nurse place the stethoscope to determine the presence of a pleural friction rub? Choose the appropriate location marked on the image. **Lower-lateral chest

Lower-lateral chest, which is the area of greatest thoracic excursion. With visceral and parietal pleural inflammation (pleurisy), a low-pitched, coarse, grating sound is heard when the client breathes, particularly when approaching the height of inspiration. Bronchial breath sounds are heard over the trachea and at the nape of the neck on either side of the vertebrae. Bronchial sounds are loud, high pitched, and hollow, with a short inspiratory phase and long expiratory phase. Bronchovesicular breath sounds are heard on either side of the sternum or between the scapulae; bronchovesicular sounds have a moderate volume and medium pitch, with equal inspiratory and expiratory phases. C is the area where vesicular breath sounds are heard. Vesicular sounds are soft and low pitched, with a long inspiratory phase and a short expiratory phase; they are heard over most lung fields.

To promote airway clearance in a patient with pneumonia, what should the nurse instruct the patient to do (select all that apply)? Maintain adequate fluid intake. Splint the chest when coughing. Maintain a 30-degree elevation. Maintain a High-Fowler's position. (UPRIGHT) Instruct patient to cough at end of exhalation.

Maintaining adequate fluid intake liquefies secretions, allowing easier expectoration. The nurse should instruct the patient to splint the chest while coughing. This will reduce discomfort and allow for a more effective cough. Coughing at the end of exhalation promotes a more effective cough. The patient should be positioned in an upright sitting position (high Fowler's) with head slightly flexed.

A client who is intubated and receiving mechanical ventilation has a problem of risk for infection. The nurse should include which measures in the care of this client? Select all that apply.

Monitor the client's temperature. Use sterile technique when suctioning. Use the closed-system method of suctioning. Monitor sputum characteristics and amounts.

A client being mechanically ventilated after experiencing a fat embolism is visibly anxious. What is the best nursing action? **Provide reassurance to the client and give small doses of morphine sulfate intravenously as prescribed.

Morphine sulfate often is prescribed for pain and anxiety in the client receiving mechanical ventilation. The nurse should speak to the client calmly and provide reassurance to the anxious client. Family members also are stressed, not just because of the complication but because of the original injury. It is not beneficial to ask the family to take on the burden of remaining with the client at all times. Succinylcholine is a neuromuscular blocker but has no antianxiety properties. Encouraging the client to sleep until arterial blood gas results improve does nothing to reassure or help the client

The nurse is caring for a client with a tracheostomy tube who is receiving mechanical ventilation. The nurse is monitoring for complications related to the tracheostomy and suspects tracheoesophageal fistula when which occurs? **Aspiration of gastric contents occurs during suctioning.

Necrosis of the tracheal wall can lead to formation of an abnormal opening between the posterior trachea and the esophagus. The opening, called a tracheoesophageal fistula, allows air to escape into the stomach, causing abdominal distention. It also causes aspiration of gastric contents.

Which test result identifies that a patient with asthma is responding to treatment? **A decreased exhaled nitric oxide

Nitric oxide levels are increased in the breath of people with asthma. A decrease in the exhaled nitric oxide concentration suggests that the treatment may be decreasing the lung inflammation associated with asthma and adherence to treatment. An increase in CO2 levels, decreased white blood cell count, and increased serum bicarbonate levels do not indicate a positive response to treatment in the asthma patient.

During admission of a patient diagnosed with non-small cell lung carcinoma, the nurse questions the patient related to a history of which risk factors for this type of cancer (select all that apply)? Asbestos exposure Exposure to uranium History of cigarette smoking

Non-small cell carcinoma is associated with cigarette smoking and exposure to environmental carcinogens, including asbestos and uranium. Chronic interstitial fibrosis is associated with the development of adenocarcinoma of the lung. Exposure to cancer-causing substances in the geographic area where the patient has lived for some time may be a risk, but not necessarily where the patient was born.

A client with chronic obstructive pulmonary disease (COPD) who is beginning oxygen therapy asks the nurse why the flow rate cannot be increased to more than 2 to 4 L/min. The nurse responds that this would be harmful because a higher oxygen flow rate could lead to which physical responses? **Decrease in the client's oxygen-based respiratory drive

Normally respiratory rate varies with the amount of carbon dioxide present in the blood. In clients with COPD, this natural regulator becomes ineffective owing to exposure to high carbon dioxide levels for prolonged periods. Thus, the level of oxygen provides the respiratory stimulus for these clients. The client with COPD should be instructed not to increase the oxygen flow rate level independently because a higher oxygen level could obliterate the respiratory drive, leading to respiratory failure.

A client with chronic obstructive pulmonary disease (COPD) who is beginning oxygen therapy asks the nurse why the flow rate cannot be increased to more than 2 to 4 L/min. The nurse responds that this would be harmful because a higher oxygen flow rate could lead to which physical responses? **Decrease in the client's oxygen-based respiratory drive

Normally respiratory rate varies with the amount of carbon dioxide present in the blood. In clients with COPD, this natural regulator becomes ineffective owing to exposure to high carbon dioxide levels for prolonged periods. Thus, the level of oxygen provides the respiratory stimulus for these clients. The client with COPD should be instructed not to increase the oxygen flow rate level independently because a higher oxygen level could obliterate the respiratory drive, leading to respiratory failure.

The most accurate measurement of the client's respiratory status if he/she has a history of heavy smoking. Check Oxygen Saturation

Oxygen saturation is a measure of the relationship between oxygen and hemoglobin; it measures the amount of oxygen available to tissues and provides a pulmonary assessment for clients at risk for hypoxia; pulse oximetry provides a continuous, noninvasive measurement of an individual's oxygen saturation. PO2 is a measure of diffusion across the alveolar membrane. This value may decrease progressively in a heavy smoker, thus it is not the most accurate measure of this client's postoperative respiratory status. Also, it requires an arterial puncture. PCO2 is an accurate measure of alveolar ventilation and is elevated with hypoventilation. This may occur in any client after abdominal surgery because deep breathing often is painful. Also, an arterial puncture is necessary to acquire a specimen for testing. Hemoglobin is a measure of the blood's capacity to transport oxygen. 29%

The nurse is preparing to care for a client who will be weaned from a cuffed tracheostomy tube. The nurse is planning to use a tracheostomy plug and plans to insert it into the opening in the outer cannula. Which nursing action is required before plugging the tube? *** Deflate the cuff on the tube.

Plugging a tracheostomy tube is usually done by inserting the tracheostomy plug (decannulation stopper) into the opening of the outer cannula. This closes off the tracheostomy, and airflow and respiration occur normally through the nose and mouth. When plugging a cuffed tracheostomy tube, the cuff must be deflated. If it remains inflated, ventilation cannot occur, and respiratory arrest could result. A tracheostomy plug could not be placed in a tracheostomy if an inner cannula was in place. The ability to swallow or speak is unrelated to weaning and plugging the tube.

The UAP asks the nurse how the respiratory therapist selects the position to be used for the procedure. The nurse responds that a position is chosen that will use gravity to help drain secretions from which primary areas? *Lobes

Postural drainage uses specific client positions that vary depending on the affected lobe or lobes. The positions usually place the head lower than the affected lung segments to facilitate drainage of secretions. Postural drainage often is done in conjunction with chest percussion for maximum effectiveness. The other options are incorrect.

The nurse determines that a client with a tracheostomy tube needs suctioning if which finding is noted? **Rhonchi are auscultated.

Presence of rhonchi is an indication that there are secretions in the large airways. The client requires suctioning if he or she cannot expectorate them. A pulse oximetry reading of 96% is an acceptable reading. A pleural friction rub is indicative of inflamed pleural surfaces. Fine crackles are indicative of air moving into previously deflated alveoli.

During the infusion of dialysate during peritoneal dialysis, the client exhibits symptoms of severe respiratory difficulty. The nurse should: **Drain the fluid from the client's peritoneal cavity

Pressure from the dialysate may cause upward displacement of the diaphragm; the dialysate should be drained from the peritoneal cavity. Additional fluid will aggravate the respiratory difficulty. The client should already be in the semi-Fowler position. Auscultation is important, but it does not alleviate the respiratory difficulty.

The nurse instructs a client regarding pursed-lip breathing and asks the client to demonstrate the breathing technique. Which observation by the nurse would indicate that the client is performing the technique correctly? **The client breathes out slowly through the mouth.

Pursed-lip breathing facilitates maximal expiration for clients with obstructive lung disease. The client should close the mouth and breathe in through the nose. The client then purses the lips and breathes out slowly through the mouth, without puffing the cheeks. The client should spend at least twice the amount of time breathing out that it took to breathe in. The client should use the abdominal muscles to assist in squeezing out all of the air. The client also is instructed to use this technique during any physical activity, inhale before beginning the activity, and exhale while performing the activity. The client is also instructed that he or she should never hold the breath.

Pneumothorax - chest tube in place

Range-of-motion exercises to the client's arm on the affected side promote aeration of the reexpanding lung and maintenance of function in the arm and shoulder. Cough suppressants are not indicated because coughing and deep breathing are encouraged to help reexpand the lung. Drainage is marked with time tapes on the side of the device. The closed system is not entered for emptying; when full, the entire device is replaced. Clamps are not necessary and should be avoided because of the danger of precipitating a tension pneumothorax.

The nurse caring for a client with a closed chest drainage system notes that the fluctuation (tidaling) in the water-seal compartment has stopped. On the basis of this assessment finding, the nurse would suspect which occurrence? **The chest tubes are obstructed.

Recalling that tidaling stops when the lung has reexpanded or if the chest drainage tubes are kinked or obstructed will direct you to the correct option.

A client who was involved in a motor vehicle collision sustained multiple internal injuries, and thoracic surgery was performed. After surgery the client has two chest tubes attached to closed chest drainage systems. Place an X on the image on the right where the nurse should palpate to identify where the complication of subcutaneous emphysema most likely may occur

Subcutaneous emphysema is the appearance of bubbles under the skin because of air trapping. **The nurse should palpate the area around a chest tube insertion site for the presence of a crackling sensation felt beneath the fingertips (crepitus), indicating that air is trapped in the tissues. Also, the chest tube insertion site should be assessed for an adequate seal.

The nurse is suctioning an unconscious client who has a tracheostomy. The nurse should perform which actions when performing this procedure? Select all that apply. *Keeping a supply of suction catheters at the bedside *Auscultating breath sounds to determine the need for suctioning. *Hyperoxygenating the client before, during, and after suctioning.

Suction equipment should be kept at the bedside of an unconscious client, regardless of whether an artificial airway is used. The nurse auscultates breath sounds every 2 to 4 hours, or more frequently as needed. The client should be hyperoxygenated before, during, and after suctioning to minimize cerebral hypoxia. Intermittent suction should be applied while the catheter is being withdrawn, not while it is being inserted. Suctioning should not be performed for longer than 10 seconds at one time to prevent cerebral hypoxia and a rise in intracranial pressure.

A nurse should determine that tracheal suctioning is needed if which is noted? **Congested breath sounds in the lung fields

Suctioning is indicated only when the client has adventitious breath sounds or has accumulation of secretions. It is not performed routinely according to time elapsed since the last suctioning (2 hours elapsed since the last suctioning). Arterial blood gas results and respiratory rate (arterial oxygen level of 90 mm Hg and respiratory rate of 18 breaths/min, up from 16 breaths/min) are not good indicators of the need for suctioning because they may be influenced by a number of other factors in addition to the need for suctioning.

A client who has undergone radical neck dissection for a tumor has a potential problem of obstruction related to postoperative edema, drainage, and secretions. To promote adequate respiratory function in this client, the nurse should implement which activities? Select all that apply.

Suctioning the client as needed Encouraging coughing every 2 hours Supporting the neck incision when the client coughs Monitoring the respiratory status frequently as prescribed.

A nurse is reinforcing instructions to a client about the use of an incentive spirometer. The nurse tells the client to sustain the inhaled breath for 3 seconds. When the client asks the nurse about the rationale for this action, the nurse explains that which is the primary benefit? **Maintain inflation of the alveoli.

Sustained inhalation when using an incentive spirometer helps maintain inflation of the terminal bronchioles and alveoli, thereby promoting better gas exchange. Routine use of devices such as an incentive spirometer can help prevent atelectasis and pneumonia in clients at risk.

Emphysemic episode is dyspneic and anxious. Teach the client to do rhythmic breathing. It is believed that the client should receive low amounts of oxygen to prevent CO2 intoxication (CO2 narcosis).

Teaching the client rhythmic breathing permits more complete exhalation and emptying of carbon dioxide from the lungs. Increasing the client's oxygen intake is contraindicated. However, the results of one recent study of clients with stable chronic obstructive pulmonary disease (COPD) indicate that the hypercarbic drive is preserved. More research is needed before this theory is applied clinically. Having the client breathe into a paper bag is contraindicated because it increases carbon dioxide retention. Checking the client's vital signs, including the blood pressure, should be part of assessment, but assessment does not decrease dyspnea.

The nurse has provided discharge instructions to the client who has had a pneumonectomy. Which statement, if made by the client, indicates an understanding of appropriate home care measures? **"I should perform arm exercises two or three times a day."

The client should be instructed to perform arm and shoulder exercises two or three times a day to prevent frozen shoulder. The client is encouraged to drink liquids to liquefy secretions, making them easier to expectorate. The client is told to expect soreness in the chest and shoulder and an altered feeling of sensation around the incision site for several weeks. It is not necessary to contact the HCP if these symptoms occur.

The nursing instructor is observing a nursing student suctioning a client through a tracheostomy tube. Which observation by the nursing instructor indicates an action by the student requiring the need for further instruction? * *Suction the ET tube

The client should be suctioned as needed. Unnecessary suctioning should be avoided because it can increase secretions and cause mechanical trauma to the tissues. The client should be hyperoxygenated with 100% oxygen before suctioning. Suction is not applied during insertion of the catheter; intermittent suction and a twirling motion of the catheter are used during withdrawal.

A client has been treated for pleural effusion with a thoracentesis. The nurse determines that this procedure has been effective if the nurse notes which assessment finding? **Absence of dyspnea

The client who has undergone thoracentesis should experience relief of the signs and symptoms experienced before the procedure. Typical signs and symptoms of pleural effusion include: dry, nonproductive cough; dyspnea (usually on exertion); decreased or absent tactile fremitus; and dull or flat percussion notes on respiratory assessment.

A client with chronic obstructive pulmonary disease (COPD) is being evaluated for lung transplantation. The nurse performs the initial physical assessment. Which findings should the nurse anticipate in this client? Select all that apply. *Dyspnea at rest *Clubbed fingers *Muscle retractions *Prolonged expiratory breathing phase

The client with COPD who is eligible for a lung transplantation has end-stage COPD and will have clinical manifestations of hypoxemia, dyspnea at rest, use of accessory muscle with retractions, clubbing, and prolonged expiratory breathing phase caused by retention of carbon dioxide.

A nurse is caring for a client diagnosed with tuberculosis (TB). Which assessments, if made by the nurse, are consistent with the usual clinical presentation of TB? Select all that apply.

The client with TB usually experiences *cough (productive or nonproductive - "morning"), fatigue, anorexia, weight loss, *dyspnea, hemoptysis, chest discomfort or pain, *chills and sweats (which may occur at night), and a low-grade fever.

The nurse provides instructions to a client after a total laryngectomy. Which statement by the client indicates a need for further instruction? **"I should use diluted alcohol on the stoma to clean it."

The client with a stoma should be instructed to wash the stoma daily with a washcloth. The client should be instructed to avoid applying diluted alcohol to a stoma because it is both drying and irritating. The client is instructed to protect the stoma from water. Soaps, cotton swabs, or tissues should be avoided because their particles may enter and obstruct the airway. A non-oil-based ointment applied to the skin around the stoma helps to prevent cracking.

A client did not seek medical treatment for a previous respiratory infection, and subsequently an empyema developed in the left lung. The nurse should assess the client for which signs and symptoms associated with this problem? **Pleural pain and fever

The client with empyema usually experiences : dyspnea, increased respiratory rate, pleural pain, night sweats, fever, anorexia, and weight loss. There is a decrease in breath sounds over the affected area, a flat sound to percussion, and decreased tactile fremitus.

The nurse is performing a cardiovascular assessment on a client. Which parameter would the nurse assess to gain the best information about the client's left-sided heart function? **Breath sounds

The client with heart failure may present with different signs and symptoms according to whether the right or the left side of the heart is failing. Peripheral edema, jugular vein distention, and hepatojugular reflux all are indicators of impaired right-sided heart function. Breath sounds are an accurate indicator of left-sided heart function.

The nurse is caring for a postoperative pneumonectomy client. Which finding on assessment of the client is an adverse sign or symptom indicating pulmonary edema? *Lung crackles

The client with pulmonary edema that developed after pneumonectomy demonstrates dyspnea, cough, frothy sputum, crackles, and possibly cyanosis. Pain with deep breathing is expected and is managed with analgesics. The client with pneumonectomy most likely will not have a chest tube because the lung has been removed. A respiratory rate of 20 breaths per minute is within normal limits.

A client tells the nurse that a health care provider has stated a diagnosis of uncomplicated or simple silicosis and asks the nurse exactly what this means. What knowledge should the nurse use in formulating a response? **The client has mild ventilation restriction and fibrosis on chest x-ray.

The client with simple silicosis may be asymptomatic or have mild ventilatory restriction and has evidence of fibrosis on chest x-ray. Pulmonary function studies reveal some decreases in vital capacity and total lung volume. Massive fibrosis is not evident at this stage.

A client diagnosed with tuberculosis is distressed over fatigue and the loss of physical stamina. What should the nurse tell the client? **This is expected, and the client should gradually increase activity as tolerated.

The client with tuberculosis has significant fatigue and loss of physical stamina. This can be very frightening for the client. The nurse teaches the client that this symptom will resolve as the therapy progresses and that the client should gradually increase activity as energy levels permit.

The nurse is assessing a client with the typical clinical manifestations of tuberculosis. The nurse should expect the client to report having fatigue and cough that have been present for how long? **Several weeks to months

The client with tuberculosis may report signs and symptoms that have been present for weeks or even months. These may include fatigue, lethargy, chest pain, anorexia and weight loss, night sweats, low-grade fever, and cough with mucoid or blood-streaked sputum. It may be the production of blood-tinged sputum that finally forces some clients to seek care.

Diagnosis of pneumonia

The client's respiratory status is the priority; *bed rest reduces oxygen demands and the *administration of oxygen increases oxygen to the alveolar capillaries; *specimens for culture and sensitivity must be obtained before the *administration of antibiotics, which prevents false microbiologic interpretation caused by the effect of the antibiotic.

When teaching the patient with chronic obstructive pulmonary disease (COPD) about smoking cessation, what information should be included related to the effects of smoking on the lungs and the increased incidence of pulmonary infections? ** Decreased alveolar macrophage function

The damage to the lungs includes alveolar macrophage dysfunction that increases the incidence of infections and thus increases patient discomfort and cost to treat the infections. Other lung damage that contributes to infections includes cilia paralysis or destruction, increased mucus secretion, and bronchospasms that lead to sputum accumulation and increased cough. The patient may already be aware of respiratory mucosa damage with hoarseness and decreased sense of smell and taste, but these do not increase the incidence of pulmonary infection.

A nurse is caring for a client with acute respiratory distress syndrome. What should the nurse expect to note in the client? **Low arterial Pao2

The earliest clinical sign of acute respiratory distress syndrome is an increased respiratory rate. Breathing becomes labored, and the client may exhibit air hunger, retractions, and cyanosis. Arterial blood gas analysis reveals increasing hypoxemia, with a Pao2 lower than 60 mm Hg.

Acute respiratory distress syndrome: **Increased respiratory rate

The earliest detectable sign of acute respiratory distress syndrome is an increased respiratory rate, which can begin from 1 to 96 hours after the initial insult to the body. This is followed by increasing : dyspnea, air hunger, retraction of accessory muscles, and cyanosis. Breath sounds may be clear or consist of fine inspiratory crackles or diffuse coarse crackles.

Purpose of chest tube with a three-chamber underwater drainage system. Control the amount of suction

The first chamber collects drainage; the second chamber provides for the underwater seal; the third chamber controls the amount of suction. The first chamber, not the third chamber, collects drainage. The second chamber, not the third chamber, provides an underwater seal. Although allowing for escape of air bubbles occurs in a three-chamber system, the purpose of the third chamber is to control suction.

A client arrives in the hospital emergency department with a bloody nose. What is the initial nursing action? *Assist the client to a sitting position with the head tilted forward.

The initial nursing action to treat the client with a bloody nose is to loosen clothing around the neck to prevent pressure on the carotid artery. The client should be assisted to a sitting position with the head tilted slightly forward, and pressure should be applied to the nares by pinching the nose toward the septum for 10 minutes. Ice packs can be applied to the nose and forehead. If these actions are not successful in controlling the bleeding, an ice collar may be applied, along with a topical vasoconstrictive medication. The health care provider also may prescribe packing of the nostrils. The client should be provided with an emesis basin and should be instructed not to swallow blood so as to reduce the risk of nausea and vomiting.

Right subclavian central venous access device inserted for total parental nutrition (TPN). Complications: Pneumothorax - Check Breath sounds

The most significant and life-threatening complication of insertion of a subclavian catheter is a pneumothorax because of the proximity of the subclavian vein and the apex of the upper lobe of the lung; a client's respiratory status always is the priority. Although a chest x-ray may be done before TPN is begun, it is not the priority immediately after insertion of the catheter. A baseline blood glucose level should be obtained before insertion of the catheter. After TPN is started, routine monitoring of blood glucose levels is important. Although assessing for a neurological deficit should be done eventually, it is not the priority at this time.

A client has a chest tube attached to a water seal drainage system. As part of routine nursing care, the nurse should ensure that which intervention is implemented? **The connection between the chest tube and the drainage system is taped, and an occlusive dressing is maintained at the insertion site.

The nurse ensures that all system connections are securely taped to prevent accidental disconnection and that an occlusive dressing is maintained at the chest tube insertion site. Continuous bubbling in the water seal chamber indicates an air leak in the system and requires immediate investigation and correction. Drainage is noted and recorded every hour during the first 24 hours after insertion and every 8 hours thereafter. The system is kept below the level of the waist. Assessment for crepitus is done once every 8 hours. Sterile water is added to the suction control chamber only as needed to replace evaporation losses.

A client is returning from surgery after a pulmonary lobectomy. Which pieces of equipment should the nurse have at the bedside? Select all that apply. ** Clamp, Vaseline gauze and Suction equipment

The nurse should anticipate that a client with a lobectomy will have a chest tube and will need suction, Vaseline gauze, and a clamp at the bedside for emergency use. The nurse would not need a code cart at the bedside unless the client was in cardiac arrest. A central line kit at the bedside does not apply to the situation.

Client who has been hospitalized and treated for tuberculosis. Teaching: *Activities should be resumed gradually. *A sputum culture is needed every 2 to 4 weeks once medication therapy is initiated. *Respiratory isolation is not necessary because family members already have been exposed. *Cover the mouth and nose when coughing or sneezing and put used tissues in plastic bags.

The nurse should provide the client and family with information about tuberculosis and allay concerns about the contagious aspect of the infection. Instruct the client to follow the medication regimen exactly as prescribed and always to have a supply of the medication on hand. Advise the client of the side effects of the medication and ways of minimizing them to ensure compliance. Reassure the client that after 2 to 3 weeks of medication therapy, it is unlikely that the client will infect anyone. Inform the client that activities should be resumed gradually and about the need for adequate nutrition and a well-balanced diet that is rich in iron, protein, and vitamin C to promote healing and prevent recurrence of infection. Inform the client and family that respiratory isolation is not necessary because family members already have been exposed. Instruct the client about thorough handwashing and to cover the mouth and nose when coughing or sneezing and to put used tissues into plastic bags. Inform the client that a sputum culture is needed every 2 to 4 weeks once medication therapy is initiated. When the results of three sputum cultures are negative, the client is no longer considered infectious and can usually return to former employment.

The nurse has assisted the health care provider and the anesthesiologist with placement of an endotracheal (ET) tube for a client in respiratory distress. What is the initial nursing action to evaluate proper ET tube placement? Use an Ambu (resuscitation) bag to ventilate the client and assess for bilateral breath sounds.

The nurse verifies the placement of an ET tube immediately by ventilating the client using an Ambu bag and by auscultating for breath sounds bilaterally, which ensures ventilation of both lungs. After this initial assessment, placement is checked radiographically. The nurse marks the ET tube at the point where it enters the nose or mouth for ongoing monitoring of correct placement, but this will not determine initial adequate placement of the ET tube.

When teaching the patient with cystic fibrosis about the diet and medications, what is the priority information to be included in the discussion? ** Pancreatic enzymes and adequate fat, calories, protein, and vitamins are needed.

The patient must take pancreatic enzymes before each meal and snack and adequate fat, calories, protein, and vitamins should be eaten. Fat-soluble vitamins are needed because they are malabsorbed with the excess mucus in the gastrointestinal system. Insulin may be needed, but there is no longer a diabetic diet, and this is not priority information at this time. DIOS develops in the terminal ileum and is treated with balanced polyethylene glycol electrolyte solution (MiraLAX) to thin bowel contents.

The physician has prescribed salmeterol (Serevent) for a patient with asthma. In reviewing the use of dry powder inhalers (DPIs) with the patient, what instructions should the nurse provide? ** "Close lips tightly around the mouthpiece and breathe in deeply and quickly."

The patient should be instructed to tightly close the lips around the mouthpiece and breathe in deeply and quickly to ensure the medicine moves down deeply into the lungs. Dry powder inhalers do not require spacer devices. The patient may not taste or sense the medicine going into the lungs

Which statement made by the patient with chronic obstructive pulmonary disease (COPD) indicates a need for further teaching regarding the use of an ipratropium inhaler? **If my breathing gets worse, I should keep taking extra puffs of the inhaler until I can breathe more easily."

The patient should not just keep taking extra puffs of the inhaler to make breathing easier. Excessive treatment could trigger paradoxical bronchospasm, which would worsen the patient's respiratory status. Rinsing the mouth after the puffs will eliminate a bad taste. Waiting 1 to 2 minutes between each puff will facilitate the effectiveness of the administration. Ipratropium is not used in an emergency for COPD.

The nurse has assisted a health care provider (HCP) with the insertion of a chest tube. The nurse monitors the client and notes fluctuation of the fluid level in the water-seal chamber after the tube is inserted. Based on this assessment, which action is most appropriate? ***Continue to monitor the client.

The presence of fluctuation of the fluid level in the water-seal chamber indicates a patent drainage system. With normal breathing, the water level rises with inspiration and falls with expiration. Fluctuation stops if the tube is obstructed, if a dependent loop exists, the suction is not working properly, or the lung has re-expanded.

The nurse teaches pursed lip breathing to a patient who is newly diagnosed with chronic obstructive pulmonary disease (COPD). The nurse reinforces that this technique will assist respiration by which mechanism? **Preventing bronchial collapse and air trapping in the lungs during exhalation

The purpose of pursed lip breathing is to slow down the exhalation phase of respiration, which decreases bronchial collapse and subsequent air trapping in the lungs during exhalation. It does not affect secretions, inhalation, or increase the rate of breathing.

When teaching the patient with bronchiectasis about manifestations to report to the health care provider, which manifestation should be included? **Increasing dyspnea

The significant clinical manifestations to report to the health care provider include increasing dyspnea, fever, chills, increased sputum production, bloody sputum, and chest pain. Although drinking at least 3 L of low-sodium fluid will help liquefy secretions to make them easier to expectorate, the health care provider does not need to be notified if the patient cannot do this one day.

Stridor

The sound is a stridor, which is a high pitched crowing sound that is often heard after traumatic intubations, and is usually heard at the larynx. Heard on inspiration, the sound often is audible without a stethoscope. A stridor is not heard at the nasal cavity, trachea, or pharynx.

Suctioning a Client's Tracheostomy: Auscultate the lungs and check the heart rate, Turn suction on to between 80 and 120 mm Hg pressure, Hyperoxygenate using 100% oxygen, Don sterile gloves, Guide the catheter into the tracheostomy tube using a sterile gloved hand.

The status of the client should be ascertained as a baseline before starting the procedure. The suction should be turned on to check its adequacy before beginning. Because oxygen will be lost during suctioning, the client should be oxygenated using 100% oxygen before initiating the procedure. Then the nurse should don sterile gloves to protect the client from infection and guide the catheter into the tracheostomy tube without using negative pressure.

Which position would best help the breathing of a client with chronic obstructive pulmonary disease (COPD)? **Tripod posiiton

The tripod position (leaning forward with elbows flexed) helps to decrease the work of breathing in clients who have severe shortness of breath caused by asthma, COPD, or respiratory failure. Positioning the arms in this manner increases the anterior-posterior diameter of the chest, thereby changing the pressures within the chest cavity. The sitting position and high-Fowler's position decrease the anterior-posterior diameter. The supine position will make breathing more difficult.

Which should the nurse do when caring for a client with chest tubes attached to a chest drainage system? **Ensure the water level in the water seal chamber is at the 2-cm level.

The water seal acts a one-way valve. It allows air and fluid to leave the pleural space but prevents re-entry of atmospheric air. The minimum amount needed is 2 cm of water. A closed chest drainage system must remain airtight at all times. The device is kept below the level of the chest. If the device is kept at the level of the chest, there can be backflow of drainage into the pleural cavity. Chest tubes should not be clamped unless specifically prescribed.

Tracheostomy tube with a high-volume, low-pressure cuff. The cuff prevents: Mucosal necrosis

These cuffs do not compress the capillary beds and thus do not cause tracheal damage. A minimal air leak is desirable to ensure the lowest possible pressure in the cuff while still maintaining placement of the tube. Secretions are increased because the cuff is a foreign body in the trachea.

The nurse is caring for a client who is postoperative day 2 open of a cholecystectomy and notes the presence of bibasilar crackles. The nurse suspects atelectasis. Which nursing actions would be appropriate for this client? Select all that apply.

This client likely has postoperative atelectasis and requires * *frequent breath sounds assessment because of the presence of adventitious breath sounds. Also, the client should ** turn, cough, and deep breathe to prevent further atelectasis and pneumonia. High-potassium diet will have no effect on the resolution of atelectasis. The client may be encouraged to increase intake to facilitate thinning of any secretions that may be present. Obtaining a chest x-ray is not a nursing action and requires a health care provider prescription.

The nurse is reviewing the ventilator settings on a client with an endotracheal tube attached to mechanical ventilation. The nurse notes that the tidal volume is set at 700 mL and determines that the tidal volume indicates which factor? **The amount of air delivered with each set breath.

Tidal volume is the amount of air delivered with each set breath on the mechanical ventilator. The respiratory rate is the number of breaths to be delivered by the ventilator. The Fio2 delivered to the client is indicated by the Fio2 indicator on the ventilator. A sigh is a breath that has a greater volume than the preset tidal volume.

The nurse is preparing to assist a client with a cuffed tracheostomy tube to eat. What intervention is the priority before the client is permitted to drink or eat? *** Inflate the cuff on the tracheostomy tube

Tracheostomy tubes are available in many sizes and are made of plastic or metal. The tubes may be reusable; however, most tubes are disposable. A tracheostomy tube may or may not have a cuff. It also may have an inner cannula. For clients receiving mechanical ventilation, a cuffed tube is used. A noncuffed tube may be used when mechanical ventilation is not required. If a client with a tracheostomy is allowed to eat and the tracheostomy has a cuff, the nurse should inflate the cuff to prevent aspiration of food or fluids. The cuff would not be deflated because of the risk of aspiration. Although the nurse would ensure that the meal tray is in a comfortable position for the client, this would not be the priority intervention. The head of the bed should always be elevated; low Fowler's position could lead to aspiration.

Pulmonary tuberculosis

Tuberculosis is an infectious disease in which recurrent fevers are present, usually in the late afternoon. Profuse diaphoresis at night (night sweats) is a classical sign of tuberculosis. Blood-tinged sputum (hemoptysis) results from pathophysiological trauma to mucous membranes. The cough is productive, not dry, because the inflammatory process causes purulent mucus.

A nurse has provided a client with tuberculosis (TB) instructions on proper handling and disposal of respiratory secretions. The nurse determines that the client demonstrates understanding of the instructions when the client makes which statement? **"I will discard used tissues in a plastic bag."

Used tissues are discarded in a plastic bag. The client with TB should wash hands carefully after each contact with respiratory secretions. Oral care should be done more frequently than once a day. The client should not only turn the head but also should cover the mouth and nose when laughing, sneezing, or coughing.

The nurse is providing instructions to a client being discharged from the hospital following removal of a chest tube that was inserted after thoracic surgery. Which statement, if made by the client, indicates a need for further teaching? ** "I should remove the chest tube site dressing as soon as I get home."

When a chest tube is removed, an occlusive dressing, consisting of petrolatum gauze covered by a dry sterile dressing, usually is placed over the chest tube site. This dressing is maintained in place until the HCP says that it may be removed. The client should avoid heavy lifting for 4 to 6 weeks after discharge to facilitate continued wound healing. The client is taught to monitor and report any signs of respiratory difficulty or any signs of infection or increased temperature.

An important nursing intervention that ensures adequate ventilatory exchange after surgery is: Positioning the client laterally with the neck extended

When positioning the client laterally with the neck extended the tongue does not obstruct the airway so that drainage of secretions, and oxygen and carbon dioxide exchange, can occur. Once the pharyngeal reflex has returned, an in-place airway can cause the client to gag and vomit, and it should be removed; the client may or may not be conscious.

A nurse is caring for a client with a tracheostomy tube attached to a ventilator. The high-pressure alarm sounds on the ventilator. The nurse should plan to perform which action? **Suction the client.

When the high-pressure alarm sounds on a ventilator, it is most likely because of an obstruction. The obstruction can be caused by the client's biting on the tube, kinking of the tubing, or mucus plugging requiring suctioning.

A nurse reads in the progress notes for a client with pneumonia that areas of the client's lungs are being perfused but are not being ventilated. The nurse interprets this occurrence correctly as the presence of which physical response? **Shunt unit

When there is no ventilation to an alveolar unit but perfusion continues, a shunt unit exists. As a result, no gas exchange occurs, and unoxgenated blood continues to circulate. Anatomical dead space normally is present in the conducting airways, where pulmonary capillaries are absent. Physiological dead space occurs with conditions such as emphysema and pulmonary embolism. Ventilation-perfusion matching refers to a matching distribution of blood flow in the pulmonary capillaries and air exchange in the alveolar units of the lungs


Related study sets

Developmental Psych- Chapter 8 Questions

View Set

Intro to Psychology 1315 Exam 2 Review (Chapters 6, 8, & 9)

View Set

anatomy and physiology guided chapter 4 cards

View Set

5 Components of Physical Fitness

View Set

OA Prep Intermediate Accounting 2

View Set

BUS 220 Chapter 1 Learn Smart Practice

View Set

Pre-Game Quiz: Intentional Torts

View Set

IS 477 Exam 2 Study Guide - Part #1

View Set